Difference between revisions of "2017 AMC 12B Problems/Problem 15"
Hydroquantum (talk | contribs) (→Problem 15) |
Hydroquantum (talk | contribs) (→Problem 15) |
||
Line 3: | Line 3: | ||
<math>\textbf{(A)}\ 9:1\qquad\textbf{(B)}\ 16:1\qquad\textbf{(C)}\ 25:1\qquad\textbf{(D)}\ 36:1\qquad\textbf{(E)}\ 37:1</math> | <math>\textbf{(A)}\ 9:1\qquad\textbf{(B)}\ 16:1\qquad\textbf{(C)}\ 25:1\qquad\textbf{(D)}\ 36:1\qquad\textbf{(E)}\ 37:1</math> | ||
− | |||
[[2017 AMC 10B Problems/Problem 19|2017 AMC 10B Problem 19 Solution]] | [[2017 AMC 10B Problems/Problem 19|2017 AMC 10B Problem 19 Solution]] |
Revision as of 16:53, 16 February 2017
Problem 15
Let be an equilateral triangle. Extend side beyond to a point so that . Similarly, extend side beyond to a point so that , and extend side beyond to a point so that . What is the ratio of the area of to the area of ?